Activities that pose risks to life are acceptable if and only if each person who bears the risks either gains some ne...

texasjohnrh on June 27, 2016

Explain

Can you explain the various answer choices and why they are wrong

Replies
Create a free account to read and take part in forum discussions.

Already have an account? log in

Mehran on July 2, 2016

@texasjohnrh absolutely! Let's take a closer look.

The stimulus here is a general principle. We know that "if and only if" is diagrammed as a double-arrow (i.e. both variables are sufficient and both variables are necessary for each other).

So we would diagram this principle as follows:

A ==> GNB or BRV

The contrapositive is:

not GNB & not BRV ==> not A

AND

GNB or BRV ==> A
not A ==> not GNB & not BRV

This is a Must Be True Principle question and we know that there are only two ways to validly apply a general principle:

(1) Invoke the sufficient condition to conclude the necessary condition.

or

(2) Invoke the sufficient condition of the contrapositive to conclude the necessary condition of the contrapositive.

Our trick that any answer choice that concludes the sufficient condition of the general principle or its contrapositive is automatically eliminated doesn't help us here since both variables are sufficient and both variables are necessary.

So let's go through the answer choices one at time:

(A) is eliminated because not replacing an older car with a new model is not an activity that poses risk to life.

(B) is concluding that smoking at an outdoor public meeting place is acceptable because the danger is minimal. However, in order to conclude that something is acceptable based on our principle, we need to invoke either "gains some net benefit that cannot be had without such risk" or "bears the risk voluntarily." So (B) would be eliminated.

(C) is concluding that the risk of fatal injury posed by riding a motorcycle without a helmet is acceptable because the motorcyclist incurs this risk willingly. Bingo! As we saw in (B), the conclusion here is acceptable and in order to conclude that something is acceptable based on our principle, we need to invoke either "gains some net benefit that cannot be had without such risk" or "bears the risk voluntarily." Here, we are told that the motorcyclist bears the risk voluntarily so (C) would be the correct answer.

(D) is concluding that motor vehicles emitting low levels of pollution is acceptable because all users of motor vehicles share the resulting benefit of inexpensive, convenient travel. However, just as we saw in (B), in order to conclude that something is acceptable based on our principle, we need to invoke either "gains some net benefit that cannot be had without such risk" or "bears the risk voluntarily." So (D) would be eliminated.

(E) is eliminated because, as stated in the answer, the national service involves no risk to life.

Hope this helps! Please let us know if you have any other questions.

secrestk on August 1, 2019

How does A not pose a risk to life? The stimulus states that the newer car has more safety features so can't we assume that choosing to keep the older car, with less safety features, inherently poses a risk?

Ravi on August 1, 2019

Great question.

(A) says, "A door-to-door salesperson declines to replace his older
car with a new model with more safety features; this is acceptable
because the decision not to replace the car is voluntary."

The problem with (A) is that there is no discussion of risk. There
isn't any indication in (A) that keeping the older car results in a
higher risk to the salesperson's life. It's possible that the new car
in (A) has more safety features but doesn't actually reduce the risk
of injury (the additional safety features could be redundant).

(C) says, "A motorcyclist rides without a helmet; the risk of fatal
injury to the motorcyclist thus incurred is acceptable because the
motorcyclist incurs this risk willingly."

(C) includes a discussion of risk, and the motorcyclist satisfies one
of the conditions in the "or" statement, as he's bearing the risk
voluntarily. Thus, his risk is acceptable according to the principle
in the stimulus, so (C) is the correct answer choice.

Does this make sense? Let us know if you have any other questions!